Mathcenter Forum  

Go Back   Mathcenter Forum > คณิตศาสตร์โอลิมปิก และอุดมศึกษา > ทฤษฎีจำนวน
สมัครสมาชิก คู่มือการใช้ รายชื่อสมาชิก ปฏิทิน ค้นหา ข้อความวันนี้ ทำเครื่องหมายอ่านทุกห้องแล้ว

ตั้งหัวข้อใหม่ Reply
 
เครื่องมือของหัวข้อ ค้นหาในหัวข้อนี้
  #31  
Old 30 สิงหาคม 2005, 00:02
tunococ tunococ ไม่อยู่ในระบบ
ลมปราณบริสุทธิ์
 
วันที่สมัครสมาชิก: 06 เมษายน 2001
ข้อความ: 118
tunococ is on a distinguished road
Post

ช่วยดูให้ด้วยนะครับ ไม่ค่อยแน่ใจ แล้วก็รู้สึกว่าวิธีไม่ค่อยดีด้วย

เนื่องจาก \(7^4 > 1599\) เลยรู้ว่า \(a_i \in \{1, 2, 3, 4, 5, 6\}\) จึงแปลงโจทย์ได้เป็น

\[
A + 16B + 81C + 256D + 625E + 1296F = 1599
\]
\[
A + B + C + D + E + F = 14
\]
โดยที่ \(A, B, C, D, E, F\) เป็นจำนวนเต็มบวกหรือ 0

พอเอาอันบนลบอันล่าง ก็จะได้

\[
15B + 80C + 255D + 624E + 1295F = 1585
\]

ตอนนี้จะสรุปได้ว่า \(E = 0\) และ \(F = 0\) หรือ \(1\)

เอา \(E\) ออก แล้วหารด้วย 5 จะได้
\[
3B + 16C + 51D + 259F = 317
\]
\[
3(B + 17D) = 317 - 16C - 259F
\]

สมมติว่า \(F = 0\) ก่อน จะได้ว่า
\[
3(B + 17D) = 317 - 16C
\]
เพื่อให้ \(3 | 317 - 16C\) จะได้ว่า C ที่เป็นไปได้คือ 2, 5, 8, 11 หรือ 14 แทนแต่ละค่าลงไปในสมการ จะได้ว่า
\[
B + 17D = 95, 79, 63, 47, 31
\]
นำไปรวมกับ \(A + B + C + D = 14\) จะรู้ว่าไม่ได้คำตอบ เพราะ \(B + D \ge 15\) ทุกกรณี
(95 = (5)17 + 10, 79 = (4)17 + 11, 63 = (3)17 + 12, 47 = (2)17 + 13, 31 = (1)17 + 14)

แล้วลองพิจารณาเมื่อ \(F = 1\) บ้าง จะเห็นว่า
\[
3(B + 17D) = 58 - 16C
\]
ค่า C ที่เป็นไปได้มีเพียงค่าเดียวคือ \(C = 1\) ซึ่งเมื่อแทนลงไป จะได้ว่า
\[
B + 17D = 14
\]
ทำให้ \(B = 14\) ในขณะที่ \(C = F = 1\) ดังนั้น เมื่อรวมกับสมการ \(A + B + C + D + F = 14\) จะไม่มีคำตอบ

สรุปว่าไม่มีคำตอบครับ ... วิธีนี้ใช้แต่แรงงานอ่า อยากได้วิธีที่ดีกว่านี้อะคับ

30 สิงหาคม 2005 00:11 : ข้อความนี้ถูกแก้ไขแล้ว 4 ครั้ง, ครั้งล่าสุดโดยคุณ tunococ
ตอบพร้อมอ้างอิงข้อความนี้
  #32  
Old 06 กันยายน 2005, 02:56
tunococ tunococ ไม่อยู่ในระบบ
ลมปราณบริสุทธิ์
 
วันที่สมัครสมาชิก: 06 เมษายน 2001
ข้อความ: 118
tunococ is on a distinguished road
Post

ที่ไม่มีคนมาเล่นต่อ เป็นเพราะผมไม่ได้ตั้งคำถามรึเปล่าอะ ... คิดคำถามไม่ค่อยเป็นซะด้วย

เอาอันนี้ละกัน โจทย์ง่าย ๆ ได้จากเพื่อน

12. กำหนดฟังก์ชัน \(A(x) = 2x \ mod \ (2n + 1)\) (a mod b = เศษจากการหาร a ด้วย b ซึ่งมีค่าตั้งแต่ 0 ถึง b - 1)

1. จงแสดงว่า \(A(x)\) เป็น permutation บนเซต \(\{1, 2, 3, ..., 2n\}\)
2. ให้ \(A^n(x) = A(A(A(...(x)))\) (composition \(n\) ครั้ง) จงพิสูจน์ว่า ถ้า \(A^m(1) = 1\) แล้ว \(A^m(x) = x\) สำหรับทุก \(x \in \{1, 2, 3, ..., 2n\}\)

Edit: ใส่เลขข้อ

01 กุมภาพันธ์ 2007 23:51 : ข้อความนี้ถูกแก้ไขแล้ว 1 ครั้ง, ครั้งล่าสุดโดยคุณ warut
ตอบพร้อมอ้างอิงข้อความนี้
  #33  
Old 06 กันยายน 2005, 10:19
nooonuii nooonuii ไม่อยู่ในระบบ
ผู้พิทักษ์กฎทั่วไป
 
วันที่สมัครสมาชิก: 25 พฤษภาคม 2001
ข้อความ: 6,408
nooonuii is on a distinguished road
Post

อ้างอิง:
ข้อความเดิมของคุณ tunococ:
ที่ไม่มีคนมาเล่นต่อ เป็นเพราะผมไม่ได้ตั้งคำถามรึเปล่าอะ ... คิดคำถามไม่ค่อยเป็นซะด้วย

เอาอันนี้ละกัน โจทย์ง่าย ๆ ได้จากเพื่อน

กำหนดฟังก์ชัน \(A(x) = 2x \ mod \ (2n + 1)\) (a mod b = เศษจากการหาร a ด้วย b ซึ่งมีค่าตั้งแต่ 0 ถึง b - 1)

1. จงแสดงว่า \(A(x)\) เป็น permutation บนเซต \(\{1, 2, 3, ..., 2n\}\)
2. ให้ \(A^n(x) = A(A(A(...(x)))\) (composition \(n\) ครั้ง) จงพิสูจน์ว่า ถ้า \(A^m(1) = 1\) แล้ว \(A^m(x) = x\) สำหรับทุก \(x \in \{1, 2, 3, ..., 2n\}\)
ขอเล่นด้วยคนครับ
ก่อนอื่นผมว่าโดเมนของ A(x) เป็นเซต {0,1,...,2n} ก็ได้ ซึ่งผมขอใช้เซตนี้เป็นโดเมนของ A(x) นะครับ

1. นิยาม B(x) = (n+1)x (mod 2n+1)
จะได้ว่า BoA(x) = x (mod 2n+1) สำหรับ x = 0,1,...,2n
ดังนั้น A(x) เป็นฟังก์ชันหนึ่งต่อหนึ่งบนเซต {0,1,2,...,2n}
แต่ A(x) นิยามบนเซตจำกัด จึงได้ว่า A(x) เป็นฟังก์ชันทั่วถึงด้วย
นั่นคือ A(x) เป็น permutation

2. Am(x) = 2mx (mod 2n+1)
จากเงื่อนไขจะได้ว่า 2m = 1 (mod 2n+1)
ดังนั้น Am(x) = 2mx = x (mod 2n+1)


รอเจ้าของโจทย์มาตรวจก่อนครับ เดี๋ยวมาโพสข้อต่อไป
__________________
site:mathcenter.net คำค้น

06 กันยายน 2005 10:29 : ข้อความนี้ถูกแก้ไขแล้ว 1 ครั้ง, ครั้งล่าสุดโดยคุณ nooonuii
ตอบพร้อมอ้างอิงข้อความนี้
  #34  
Old 06 กันยายน 2005, 20:54
tunococ tunococ ไม่อยู่ในระบบ
ลมปราณบริสุทธิ์
 
วันที่สมัครสมาชิก: 06 เมษายน 2001
ข้อความ: 118
tunococ is on a distinguished road
Post

ถูกแล้วล่ะครับ

อ้อ แล้วที่ผมกำหนดโดเมนเป็น \(\{1, 2, 3, ..., 2n\}\) น่ะ เพราะว่าจริง ๆ มันมีโจทย์อีกข้อด้วย :P แต่ไม่ได้ถามเพราะมันเป็นโจทย์ algorithm น่ะครับ จริง ๆ A เป็น permutation บนโดเมน \(\{0, 1, 2, ..., 2n\}\) อย่างที่ว่าแหละครับ
ตอบพร้อมอ้างอิงข้อความนี้
  #35  
Old 07 กันยายน 2005, 04:07
nooonuii nooonuii ไม่อยู่ในระบบ
ผู้พิทักษ์กฎทั่วไป
 
วันที่สมัครสมาชิก: 25 พฤษภาคม 2001
ข้อความ: 6,408
nooonuii is on a distinguished road
Post

มาแล้วครับโจทย์ของผม ไม่ยากครับ แถมแอบๆไปทางพีชคณิตมากกว่า

13. ให้ n เป็นจำนวนนับ และ p(x) = x2 + ax + b เมื่อ a,b เป็นจำนวนเต็ม
จงพิสูจน์ว่ามีจำนวนเต็ม k ซึ่งทำให้
\[ \Large{p(n)p(n+1) = p(k)} \]

Edit: ใส่เลขข้อ
__________________
site:mathcenter.net คำค้น

01 กุมภาพันธ์ 2007 23:52 : ข้อความนี้ถูกแก้ไขแล้ว 1 ครั้ง, ครั้งล่าสุดโดยคุณ warut
ตอบพร้อมอ้างอิงข้อความนี้
  #36  
Old 09 กันยายน 2005, 01:20
nongtum's Avatar
nongtum nongtum ไม่อยู่ในระบบ
ผู้พิทักษ์กฎทั่วไป
 
วันที่สมัครสมาชิก: 10 เมษายน 2005
ข้อความ: 3,246
nongtum is on a distinguished road
Icon16

อ้างอิง:
ข้อความเดิมของคุณ nooonuii:

ให้ n เป็นจำนวนนับ และ p(x) = x2 + ax + b เมื่อ a,b เป็นจำนวนเต็ม
จงพิสูจน์ว่ามีจำนวนเต็ม k ซึ่งทำให้
\[ \Large{p(n)p(n+1) = p(k)} \]
เป็นการเพียงพอที่จะแสดงว่า \(4p(n)p(n+1)+a^2-4b\) เป็นกำลังสองสมบูรณ์ (ซึ่งกำลังสองที่ว่ามีค่าเท่ากับ (2k+a)2)
จากรูปด้านล่าง เราจะได้ k=n2+an+n+b เป็นจำนวนเต็มที่ต้องการหา

ปล. คงไม่ต้องบอกนะครับว่าคิดยังไง เพราะข้อนี้ลองทดด้วยมือมาสองวันแล้วแยก factor ไม่ออก
__________________
คนไทยร่วมใจอย่าใช้ภาษาวิบัติ
ฝึกพิมพ์สัญลักษณ์สักนิด ชีวิต(คนตอบและคนถาม)จะง่ายขึ้นเยอะ (จริงๆนะ)

Stay Hungry. Stay Foolish.
ตอบพร้อมอ้างอิงข้อความนี้
  #37  
Old 09 กันยายน 2005, 03:47
nongtum's Avatar
nongtum nongtum ไม่อยู่ในระบบ
ผู้พิทักษ์กฎทั่วไป
 
วันที่สมัครสมาชิก: 10 เมษายน 2005
ข้อความ: 3,246
nongtum is on a distinguished road
Cool

เกือบลืมตั้งคำถามข้อต่อไป

14. จงหาจำนวนนับ k ทั้งหมดที่ทำให้ 1k+9k+10k=5k+6k+11k
__________________
คนไทยร่วมใจอย่าใช้ภาษาวิบัติ
ฝึกพิมพ์สัญลักษณ์สักนิด ชีวิต(คนตอบและคนถาม)จะง่ายขึ้นเยอะ (จริงๆนะ)

Stay Hungry. Stay Foolish.
ตอบพร้อมอ้างอิงข้อความนี้
  #38  
Old 09 กันยายน 2005, 12:02
nooonuii nooonuii ไม่อยู่ในระบบ
ผู้พิทักษ์กฎทั่วไป
 
วันที่สมัครสมาชิก: 25 พฤษภาคม 2001
ข้อความ: 6,408
nooonuii is on a distinguished road
Post

อ้างอิง:
ข้อความเดิมของคุณ nooonuii:

ให้ n เป็นจำนวนนับ และ p(x) = x2 + ax + b เมื่อ a,b เป็นจำนวนเต็ม
จงพิสูจน์ว่ามีจำนวนเต็ม k ซึ่งทำให้
\[ \Large{p(n)p(n+1) = p(k)} \]
คุณ Nongtum ความพยายามเป็นเลิศจริงๆครับ
__________________
site:mathcenter.net คำค้น
ตอบพร้อมอ้างอิงข้อความนี้
  #39  
Old 11 กันยายน 2005, 03:03
tunococ tunococ ไม่อยู่ในระบบ
ลมปราณบริสุทธิ์
 
วันที่สมัครสมาชิก: 06 เมษายน 2001
ข้อความ: 118
tunococ is on a distinguished road
Post

โจทย์ของ nongtum ตอบว่า 2 กับ 4 ครับ

ผมคิดโดยแทนค่าทั้งหมดที่เป็นไปได้น่ะครับ แล้วมันได้แค่ 2 กับ 4

วิธีหาค่าที่เป็นไปได้ 2 เซตของผม เอามา intersect กัน คือ

1. คิดที่ mod 3 จะรู้ว่า k ต้องเป็นเลขคู่

2. หา upperbound ของ k

เนื่องจาก \(1^k < 5^k + 6^k\) อยู่แล้ว ต้องการหาค่า n ที่ทำให้ \(9^n + 10^n < 11^n\) เพื่อจะสรุปว่าฝั่งซ้าย น้อยกว่าฝั่งกว่า เมื่อ \(k \ge n\)

ลองแทนค่าเอาจะเห็นว่า \(n = 5\) เป็นค่า \(n\) ที่น้อยที่สุด ดังนั้น ค่า k มากที่สุดที่เป็นคำตอบได้คือ 4 คำตอบที่เป็นไปได้ก็เลยมีแค่ 2 กับ 4 เท่านั้น โชคดีที่มันใช้ได้ทั้งคู่

ถ้าไม่แทนค่าเอา จะคิดเอาก็ได้ครับ วิธีขี้เกียจหน่อยก็จะให้ค่า n = 8 ซึ่งก็ไม่มากเกินแรงเครื่องคิดเลขจิ้มครับ เพราะตัวที่ต้องทดลองเพิ่มมีตัวเดียวคือ k = 6 (ทดลองที่ mod 6 ก็จะเห็นว่าไม่ได้)

วิธีขี้เกียจของผมก็คือ

\[
\begin{eqnarray}
9^n + 10^n \le 10^n + 10^n = 2 \times 10^n & < & 11^n \\
\log 2 + n \log 10 & < & n \log 11 \\
\frac{\log 2}{\log 11 - 1} & < & n \\
7 & < & n
\end{eqnarray}
\]
ไม่รู้จะถามอะไรต่อดีอะ ...

11 กันยายน 2005 03:16 : ข้อความนี้ถูกแก้ไขแล้ว 3 ครั้ง, ครั้งล่าสุดโดยคุณ tunococ
ตอบพร้อมอ้างอิงข้อความนี้
  #40  
Old 25 กันยายน 2005, 03:22
warut warut ไม่อยู่ในระบบ
กระบี่ไร้สภาพ
 
วันที่สมัครสมาชิก: 24 พฤศจิกายน 2001
ข้อความ: 1,627
warut is on a distinguished road
Smile

อ้างอิง:
ข้อความเดิมของคุณ gools:
11. จงหาจำนวนคำตอบที่เป็นจำนวนเต็มบวกของสมการ
\[a_1^4+a_2^4+...+a_{14}^4=1,599\]
ถ้า a เป็นจำนวนคู่ แล้ว \(a^4\equiv0\pmod{16}\)
ถ้า a เป็นจำนวนคี่ แล้ว \(a^4\equiv1\pmod{16}\)
ดังนั้น least residue modulo 16 ของ \(a_1^4+a_2^4+...+a_{14}^4\) จึงมีค่าอยู่ระหว่าง 0 ถึง 14
แต่ least residue modulo 16 ของ 1599 มีค่าเท่ากับ 15 ดังนั้นสมการนี้จึงไม่มีคำตอบครับ

Edit: ใส่เลขข้อ

01 กุมภาพันธ์ 2007 23:55 : ข้อความนี้ถูกแก้ไขแล้ว 1 ครั้ง, ครั้งล่าสุดโดยคุณ warut
ตอบพร้อมอ้างอิงข้อความนี้
  #41  
Old 25 กันยายน 2005, 20:07
tunococ tunococ ไม่อยู่ในระบบ
ลมปราณบริสุทธิ์
 
วันที่สมัครสมาชิก: 06 เมษายน 2001
ข้อความ: 118
tunococ is on a distinguished road
Post

เย่ ๆ มีคนมาตอบวิธีดี ๆ ให้แล้ว ขอบคุณครับ
ตอบพร้อมอ้างอิงข้อความนี้
  #42  
Old 26 กันยายน 2005, 23:04
nongtum's Avatar
nongtum nongtum ไม่อยู่ในระบบ
ผู้พิทักษ์กฎทั่วไป
 
วันที่สมัครสมาชิก: 10 เมษายน 2005
ข้อความ: 3,246
nongtum is on a distinguished road
Post

กระทู้นี้เงียบสนิท(ข้อ 14 ถูกแล้วครับ) เอาเป็นว่าขอตั้งคำถามอีกข้อละกัน

15. จำนวน \(1991^{1991}+1992^{1992}+1993^{1993}+1994^{1994}+1995^{1995}+1996^{1996} \) เป็นจำนวนจัตุรัสหรือไม่
__________________
คนไทยร่วมใจอย่าใช้ภาษาวิบัติ
ฝึกพิมพ์สัญลักษณ์สักนิด ชีวิต(คนตอบและคนถาม)จะง่ายขึ้นเยอะ (จริงๆนะ)

Stay Hungry. Stay Foolish.
ตอบพร้อมอ้างอิงข้อความนี้
  #43  
Old 27 กันยายน 2005, 10:03
nooonuii nooonuii ไม่อยู่ในระบบ
ผู้พิทักษ์กฎทั่วไป
 
วันที่สมัครสมาชิก: 25 พฤษภาคม 2001
ข้อความ: 6,408
nooonuii is on a distinguished road
Post

อ้างอิง:
ข้อความเดิมของคุณ nongtum:
กระทู้นี้เงียบสนิท(ข้อ 14 ถูกแล้วครับ) เอาเป็นว่าขอตั้งคำถามอีกข้อละกัน

15. จำนวน \(1991^{1991}+1992^{1992}+1993^{1993}+1994^{1994}+1995^{1995}+1996^{1996} \) เป็นจำนวนจัตุรัสหรือไม่
__________________
site:mathcenter.net คำค้น
ตอบพร้อมอ้างอิงข้อความนี้
  #44  
Old 29 กันยายน 2005, 05:11
nooonuii nooonuii ไม่อยู่ในระบบ
ผู้พิทักษ์กฎทั่วไป
 
วันที่สมัครสมาชิก: 25 พฤษภาคม 2001
ข้อความ: 6,408
nooonuii is on a distinguished road
Post

โจทย์ข้อนี้ผมไม่แน่ใจว่าจะใช้ความรู้ทฤษฎีจำนวนพื้นฐานแก้ได้รึเปล่าครับ ยังไม่เคยลองเหมือนกัน ข้อนี้เป็นการบ้านวิชา Dynamical Systems ที่ผมกำลังเรียนอยู่ครับ

16. จงพิสูจน์ว่ามีจำนวนเต็มบวก n ซึ่งทำให้ 2n ขึ้นต้นด้วย 2548 เมื่อเขียนเป็นเลขฐานสิบ
__________________
site:mathcenter.net คำค้น
ตอบพร้อมอ้างอิงข้อความนี้
  #45  
Old 30 กันยายน 2005, 08:10
nongtum's Avatar
nongtum nongtum ไม่อยู่ในระบบ
ผู้พิทักษ์กฎทั่วไป
 
วันที่สมัครสมาชิก: 10 เมษายน 2005
ข้อความ: 3,246
nongtum is on a distinguished road
Post

ข้อ 16 ของคุณ nooonuii ผมขอเสนอ'แนวคิด'ดังนี้ครับ ไม่รู้ว่าจะพอไหวไหม...

หากมี n ที่ว่าจริง จะต้องมีจำนวนเต็มบวก d ที่ทำให้ \(2548\cdot10^d<2^n<2549\cdot10^d\) เนื่องจาก \(\log_22549-\log_22548\approx0.0006...\) จะได้ว่า \(1-(เลขหลังจุดทศนิยมของ\log_{2}(2548\cdot10^d))<0.0006\) ซึ่งสามารถแก้ได้หลายแบบ ทั้งแทนค่าแล้วดูค่า d ที่ใกล้กับจำนวนเต็มหรือวิธีอื่นๆซึ่งจะไม่กล่าวในที่นี้ (โจทย์ถามแค่ว่ามีหรือไม่ ไม่ได้ถามว่าหาอย่างไร)

หมายเหตุ: เท่าที่ลองใช้คอมคิด พบว่า d>300 ส่วนที่ว่าจะมี d ที่สอดคล้องเงื่อนไขมากกว่าหนึ่งตัวหรือไม่ไม่ได้คิดครับ(แต่คาดว่ามีมากกว่าหนึ่งแน่ๆ)
__________________
คนไทยร่วมใจอย่าใช้ภาษาวิบัติ
ฝึกพิมพ์สัญลักษณ์สักนิด ชีวิต(คนตอบและคนถาม)จะง่ายขึ้นเยอะ (จริงๆนะ)

Stay Hungry. Stay Foolish.
ตอบพร้อมอ้างอิงข้อความนี้
ตั้งหัวข้อใหม่ Reply


หัวข้อคล้ายคลึงกัน
หัวข้อ ผู้ตั้งหัวข้อ ห้อง คำตอบ ข้อความล่าสุด
ปัญหาชิงรางวัลข้อที่ 23: Number Theory once more warut คณิตศาสตร์อุดมศึกษา 17 28 ธันวาคม 2011 20:38
ช่วยคิดหน่อยครับ เกี่ยวกับ Number Theory kanji ทฤษฎีจำนวน 0 08 กันยายน 2006 18:22
ปัญหาชิงรางวัลข้อที่ 5: From Number Theory Marathon warut คณิตศาสตร์อุดมศึกษา 9 17 มกราคม 2006 18:47
ปัญหา Number Theory kanji ทฤษฎีจำนวน 4 16 พฤศจิกายน 2005 20:30
ขอลองตั้งคำถามบ้างครับ (Number theory) Nay ทฤษฎีจำนวน 3 15 พฤษภาคม 2005 13:40

เครื่องมือของหัวข้อ ค้นหาในหัวข้อนี้
ค้นหาในหัวข้อนี้:

ค้นหาขั้นสูง

กฎการส่งข้อความ
คุณ ไม่สามารถ ตั้งหัวข้อใหม่ได้
คุณ ไม่สามารถ ตอบหัวข้อได้
คุณ ไม่สามารถ แนบไฟล์และเอกสารได้
คุณ ไม่สามารถ แก้ไขข้อความของคุณเองได้

vB code is On
Smilies are On
[IMG] code is On
HTML code is Off
ทางลัดสู่ห้อง


เวลาที่แสดงทั้งหมด เป็นเวลาที่ประเทศไทย (GMT +7) ขณะนี้เป็นเวลา 01:35


Powered by vBulletin® Copyright ©2000 - 2024, Jelsoft Enterprises Ltd.
Modified by Jetsada Karnpracha